The Student Room Group

Maths question

Hi guys, so I understand i) and ii) and for iii) I understand the table drawn in the mark scheme and the fact that (1 + 1/n )^n = 1 +1 + ((n-1)/2n) + ((n-1)(n-2)/6n^2) + ((n-1)(n-2(n-3)/24n^3) where:

1 is the first term (so the first term is not when n=1?)

1 is the second term

((n-1)/2n) is the third term

((n-1)(n-2)/6n^2) is the fourth term

((n-1)(n-2(n-3)/24n^3) is the fifth term

However, I don’t understand the question and the rest of the mark scheme explanation for part iii)

Also, for part iv) and iii) is there a quicker way to fill out the table than the long way of subbing for every n value? And I don’t get the part in the mark scheme about constant e for iv).

Can someone please help? I have attached both the question and the mark scheme
19.PNG
Attachment not found
(edited 6 years ago)
I think for part (iii) you are supposed to infer that the five-term approximation tends to 1 + 1 + 1/2 + 1/6 + 1/24 for very large values of n. There may be some words missing in the mark scheme.

In part (iv) the thing about e is that if you have come across the numerical value of e before, you may recognise something rather close to it it in the spreadsheet for very large values on n.

As for filling out the tables, it's easy enough in a spreadsheet as you can simply copy the relevant spreadsheet formula from one cell to another. On a calculator like the Casio FX-991EX you can save a bit of time by using Table Mode, or in normal mode by typing in expressions for the third, fourth and fifth terms or the series separate by "ALPHA :" symbols then CALC'ing for different numerical values.
Reply 2
Original post by old_engineer
I think for part (iii) you are supposed to infer that the five-term approximation tends to 1 + 1 + 1/2 + 1/6 + 1/24 for very large values of n. There may be some words missing in the mark scheme.

In part (iv) the thing about e is that if you have come across the numerical value of e before, you may recognise something rather close to it it in the spreadsheet for very large values on n.

As for filling out the tables, it's easy enough in a spreadsheet as you can simply copy the relevant spreadsheet formula from one cell to another. On a calculator like the Casio FX-991EX you can save a bit of time by using Table Mode, or in normal mode by typing in expressions for the third, fourth and fifth terms or the series separate by "ALPHA :" symbols then CALC'ing for different numerical values.


Thanks very much for your reply
Sorry I have just attached here the correct version:
20.png
And how did you infer for part iii) that the five-term approximation tends to 1 + 1 + 1/2 + 1/6 + 1/24 for very large values of n?
Original post by sienna2266
Thanks very much for your reply
Sorry I have just attached here the correct version:
And how did you infer for part iii) that the five-term approximation tends to 1 + 1 + 1/2 + 1/6 + 1/24 for very large values of n?


For a fraction like (n1)(n2)6n2\dfrac{(n-1)(n-2)}{6n^2} it is easy to see that if you expand the numerator and take the highest order term, which is n2n^2 here, then divide each term of the fraction by it, then you will be left with 13n+2n26\dfrac{1-\frac{3}{n}+\frac{2}{n^2}}{6}. When nn \rightarrow \infty, each little fraction with an nn in the denominator will tend to 0, hence the limit is 16\frac{1}{6}
Original post by sienna2266
And how did you infer for part iii) that the five-term approximation tends to 1 + 1 + 1/2 + 1/6 + 1/24 for very large values of n?


For example ((n - 1)(n - 2)) / 6n^2 = (n^2 - 3n + 2) / 6n^2. For very large n, the "3n" and "2" terms will become vanishingly small compared to the x^2 terms. So (n^2 - 3n + 2) / 6n^2 --> n^2 / 6n^2 --> 1 / 6.
Similar reasoning for the other terms.
(edited 6 years ago)
Reply 5
Original post by RDKGames
For a fraction like (n1)(n2)6n2\dfrac{(n-1)(n-2)}{6n^2} it is easy to see that if you expand the numerator and take the highest order term, which is n2n^2 here, then divide each term of the fraction by it, then you will be left with 13n+2n26\dfrac{1-\frac{3}{n}+\frac{2}{n^2}}{6}. When nn \rightarrow \infty, each little fraction with an nn in the denominator will tend to 0, hence the limit is 16\frac{1}{6}


Thanks very much -the calculations side of this question makes sense now.
I am a bit confused with the wording of the mark scheme:
22.PNG
As n get bigger, all the “ratios” of terms involving n in the approximation get closer and closer to 1, so in the limit the sequence of the Professor’s calculations seem likely to tend to 2 17/24.

Doesn’t it get closer and closer to 0 b/c i.e. ((n-1)(n-2))/6 = (1- (3/n)-(2/n^2))/6

-3/n and 2/n^2 get closer and closer to 0 as n-->infinity so 1-0/6 --> 1/6

More like the limit of the sequence is 2 17/24 or the sequence tends to 2 17/24 not in the limit the sequence of the Professor’s calculations seem likely to tend to 2 17/24.”

Could you please let me know if I have the wrong understanding for the wording of the mark scheme answer?
Reply 6
Original post by old_engineer
For example ((n - 1)(n - 2)) / 6n^2 = (n^2 - 3n + 2) / 6n^2. For very large n, the "3n" and "2" terms will become vanishingly small compared to the x^2 terms. So (n^2 - 3n + 2) / 6n^2 --> n^2 / 6n^2 --> 1 / 6.
Similar reasoning for the other terms.


Thanks very much -the calculations side of this question makes sense now.
I am a bit confused with the wording of the mark scheme:
22.PNG
As n get bigger, all the “ratios” of terms involving n in the approximation get closer and closer to 1, so in the limit the sequence of the Professor’s calculations seem likely to tend to 2 17/24.

Doesn’t it get closer and closer to 0 b/c i.e. ((n-1)(n-2))/6 = (1- (3/n)-(2/n^2))/6

-3/n and 2/n^2 get closer and closer to 0 as n-->infinity so 1-0/6 --> 1/6

More like the limit of the sequence is 2 17/24 or the sequence tends to 2 17/24 not in the limit the sequence of the Professor’s calculations seem likely to tend to 2 17/24.”

Could you please let me know if I have the wrong understanding for the wording of the mark scheme answer?
Original post by sienna2266
Thanks very much -the calculations side of this question makes sense now.
I am a bit confused with the wording of the mark scheme:
22.PNG
As n get bigger, all the “ratios” of terms involving n in the approximation get closer and closer to 1, so in the limit the sequence of the Professor’s calculations seem likely to tend to 2 17/24.

Doesn’t it get closer and closer to 0 b/c i.e. ((n-1)(n-2))/6 = (1- (3/n)-(2/n^2))/6

-3/n and 2/n^2 get closer and closer to 0 as n-->infinity so 1-0/6 --> 1/6

More like the limit of the sequence is 2 17/24 or the sequence tends to 2 17/24 not in the limit the sequence of the Professor’s calculations seem likely to tend to 2 17/24.”

Could you please let me know if I have the wrong understanding for the wording of the mark scheme answer?


I also found the wording in the mark scheme a little odd. Here's a stab at what the words might mean: "as n gets bigger, the ratio of the value of any particular term in the series for n = (k+1) to the value for n = k approaches 1, that is to say the value of the term changes more slowly as n gets larger and in fact tends towards a fixed value for very large n".
Reply 8
Original post by old_engineer
I also found the wording in the mark scheme a little odd. Here's a stab at what the words might mean: "as n gets bigger, the ratio of the value of any particular term in the series for n = (k+1) to the value for n = k approaches 1, that is to say the value of the term changes more slowly as n gets larger and in fact tends towards a fixed value for very large n".


Thanks very much for your help- I am still a little confused with the "...get closer and closer to 1.." bit :frown:

I guess its just a weird mark scheme
Reply 9
Original post by RDKGames
For a fraction like (n1)(n2)6n2\dfrac{(n-1)(n-2)}{6n^2} it is easy to see that if you expand the numerator and take the highest order term, which is n2n^2 here, then divide each term of the fraction by it, then you will be left with 13n+2n26\dfrac{1-\frac{3}{n}+\frac{2}{n^2}}{6}. When nn \rightarrow \infty, each little fraction with an nn in the denominator will tend to 0, hence the limit is 16\frac{1}{6}


Thanks very much -the calculations side of this question makes sense now.
I am a bit confused with the wording of the mark scheme:

As n get bigger, all the “ratios” of terms involving n in the approximation get closer and closer to 1, so in the limit the sequence of the Professor’s calculations seem likely to tend to 2 17/24.

Doesn’t it get closer and closer to 0 b/c i.e. ((n-1)(n-2))/6 = (1- (3/n)-(2/n^2))/6

-3/n and 2/n^2 get closer and closer to 0 as n-->infinity so 1-0/6 --> 1/6

More like the limit of the sequence is 2 17/24 or the sequence tends to 2 17/24 not in the limit the sequence of the Professor’s calculations seem likely to tend to 2 17/24.”

Could you please let me know if I have the wrong understanding for the wording of the mark scheme answer?
Original post by sienna2266
Thanks very much -the calculations side of this question makes sense now.
I am a bit confused with the wording of the mark scheme:

As n get bigger, all the “ratios” of terms involving n in the approximation get closer and closer to 1, so in the limit the sequence of the Professor’s calculations seem likely to tend to 2 17/24.

Doesn’t it get closer and closer to 0 b/c i.e. ((n-1)(n-2))/6 = (1- (3/n)-(2/n^2))/6If you look at the (n1)(n2)6n2\dfrac{(n-1)(n-2)}{6n^2} term, for example, you can write this as:

(n1n)(n2n)16\left(\dfrac{n-1}{n}\right) \left(\dfrac{n-2}{n}\right) \dfrac{1}{6}

The two items in brackets are the ratios (n-1)/n and (n-2)/n, both of which tend to 1 as n tends to infinity.

(Your own argument is also valid (but different), but you've had to multiply out for it, and obviously that will get painful when you have enough terms multiplied together).

Edit: FWIW, I'd say this is a somewhat sloppily worded question and solution - it's unfortunate that the person writing it doesn't seem quite able to decide whether readers are supposed to understand limits or not, which I think makes things more confusing. Where's the Q from?
(edited 6 years ago)
Original post by DFranklin
If you look at the (n1)(n2)6n2\dfrac{(n-1)(n-2)}{6n^2} term, for example, you can write this as:

(n1n)(n2n)16\left(\dfrac{n-1}{n}\right) \left(\dfrac{n-2}{n}\right) \dfrac{1}{6}

The two items in brackets are the ratios (n-1)/n and (n-2)/n, both of which tend to 1 as n tends to infinity.

(Your own argument is also valid (but different), but you've had to multiply out for it, and obviously that will get painful when you have enough terms multiplied together).

Edit: FWIW, I'd say this is a somewhat sloppily worded question and solution - it's unfortunate that the person writing it doesn't seem quite able to decide whether readers are supposed to understand limits or not, which I think makes things more confusing. Where's the Q from?


Thanks so much -this makes sense!!

It's from Mei question papers :smile: BTW this is Core 1
(edited 6 years ago)
Original post by sienna2266
It's from Mei question papers :smile: BTW this is Core 1


Are you self-teaching from these resources?
Original post by RDKGames
Are you self-teaching from these resources?


Well kind of, my teachers are not that great.
Original post by sienna2266
Well kind of, my teachers are not that great.


From what I've seen with your questions, these resources aren't that great either - they're not as clear, precise and explained as they should be.
Original post by RDKGames
From what I've seen with your questions, these resources aren't that great either - they're not as clear, precise and explained as they should be.


True :biggrin: But I try to do them for extra understanding or in case something like it ever comes up in the exams ... C4 comprehension paper...

Quick Reply

Latest